Difference between revisions of "1997 AIME Problems/Problem 15"

m
(solution by elemennop)
Line 1: Line 1:
 
== Problem ==
 
== Problem ==
The sides of rectangle <math>ABCD</math> have lengths <math>10</math> and <math>11</math>. An equilateral triangle is drawn so that no point of the triangle lies outside <math>ABCD</math>. The maximum possible area of such a triangle can be written in the form <math>p\sqrt{q}-r</math>, where <math>p</math>, <math>q</math>, and <math>r</math> are positive integers, and <math>q</math> is not divisible by the square of any prime number. Find <math>p+q+r</math>.
+
The sides of [[rectangle]] <math>ABCD</math> have lengths <math>10</math> and <math>11</math>. An [[equilateral triangle]] is drawn so that no point of the triangle lies outside <math>ABCD</math>. The maximum possible [[area]] of such a triangle can be written in the form <math>p\sqrt{q}-r</math>, where <math>p</math>, <math>q</math>, and <math>r</math> are positive integers, and <math>q</math> is not divisible by the square of any prime number. Find <math>p+q+r</math>.
  
 
== Solution ==
 
== Solution ==
{{solution}}
+
Consider points on the [[complex plane]] <math>A (0,0),\ B (11,0),\ C (11,10),\ D (0,10)</math>. Since the rectangle is quite close to a square, we figure that the area of the equilateral triangle is maximized when a vertex of the triangle coincides with that of the rectangle. Set one [[vertex]] of the triangle at <math>A</math>, and the other two points <math>E</math> and <math>F</math> on <math>BC</math> and <math>CD</math>, respectively. Let <math>E (11,a)</math> and <math>F (b, 10)</math>. Since it's equilateral, then <math>E\cdot\text{cis}60^{\circ} = F</math>, so <math>(11 + ai)(\frac {1}{2} + \frac {\sqrt {3}}{2}i) = b + 10i</math>, and expanding we get <math>(\frac {11}{2} - \frac {a\sqrt {3}}{2}) + (\frac {11\sqrt {3}}{2} + \frac {a}{2})i = b + 10i</math>.
 +
 
 +
We can then set the real and imaginary parts equal, and solve for <math>(a,b) = (20 - 11\sqrt {3},22 - 10\sqrt {3})</math>. We can then use [[Shoelace Theorem]] to find the area to be <math>221\sqrt {3} - 330</math>, So <math>p + q + r = 221 + 3 + 330 = 554</math>.
  
 
== See also ==
 
== See also ==
 
{{AIME box|year=1997|num-b=14|after=Last Question}}
 
{{AIME box|year=1997|num-b=14|after=Last Question}}
 +
 +
[[Category:Intermediate Geometry Problems]]

Revision as of 20:38, 24 November 2007

Problem

The sides of rectangle $ABCD$ have lengths $10$ and $11$. An equilateral triangle is drawn so that no point of the triangle lies outside $ABCD$. The maximum possible area of such a triangle can be written in the form $p\sqrt{q}-r$, where $p$, $q$, and $r$ are positive integers, and $q$ is not divisible by the square of any prime number. Find $p+q+r$.

Solution

Consider points on the complex plane $A (0,0),\ B (11,0),\ C (11,10),\ D (0,10)$. Since the rectangle is quite close to a square, we figure that the area of the equilateral triangle is maximized when a vertex of the triangle coincides with that of the rectangle. Set one vertex of the triangle at $A$, and the other two points $E$ and $F$ on $BC$ and $CD$, respectively. Let $E (11,a)$ and $F (b, 10)$. Since it's equilateral, then $E\cdot\text{cis}60^{\circ} = F$, so $(11 + ai)(\frac {1}{2} + \frac {\sqrt {3}}{2}i) = b + 10i$, and expanding we get $(\frac {11}{2} - \frac {a\sqrt {3}}{2}) + (\frac {11\sqrt {3}}{2} + \frac {a}{2})i = b + 10i$.

We can then set the real and imaginary parts equal, and solve for $(a,b) = (20 - 11\sqrt {3},22 - 10\sqrt {3})$. We can then use Shoelace Theorem to find the area to be $221\sqrt {3} - 330$, So $p + q + r = 221 + 3 + 330 = 554$.

See also

1997 AIME (ProblemsAnswer KeyResources)
Preceded by
Problem 14
Followed by
Last Question
1 2 3 4 5 6 7 8 9 10 11 12 13 14 15
All AIME Problems and Solutions